Why does this integral equal $0$?











up vote
11
down vote

favorite
5












In this question, the OP poses the following definite integral, which just happens to vanish:
$$int_1^sqrt2 frac{1}{x}lnbigg(frac{2-2x^2+x^4}{2x-2x^2+x^3}bigg)dx=0$$
As noticed by one commenter to the question, the only zero of the integrand is at $x=sqrt[3]{2}$, meaning that the integral of the integrand from $x=1$ to $x=sqrt[3]{2}$ is the additive inverse of the integral of the integrand from $x=sqrt[3]{2}$ to $x=sqrt{2}$.



This suggests some sort of symmetry obtainable by a substitution, but I cannot find an appropriate substitution or cancellation. It seems like the answer should be much simpler than those posted to the linked question.



Any ideas?



EDIT: I believe that this more general integral also vanishes:
$$int_1^{sqrt{t}}frac{1}{x}lnbigg(frac{t-sx^2+x^4}{tx-sx^2+x^3}bigg)dx=0$$










share|cite|improve this question
























  • @Rócherz Of course it isn't symmetric about $sqrt[3]{2}$; I simply mean that perhaps an appropriate substitution might lead to a cancellation.
    – Frpzzd
    Nov 20 at 0:47















up vote
11
down vote

favorite
5












In this question, the OP poses the following definite integral, which just happens to vanish:
$$int_1^sqrt2 frac{1}{x}lnbigg(frac{2-2x^2+x^4}{2x-2x^2+x^3}bigg)dx=0$$
As noticed by one commenter to the question, the only zero of the integrand is at $x=sqrt[3]{2}$, meaning that the integral of the integrand from $x=1$ to $x=sqrt[3]{2}$ is the additive inverse of the integral of the integrand from $x=sqrt[3]{2}$ to $x=sqrt{2}$.



This suggests some sort of symmetry obtainable by a substitution, but I cannot find an appropriate substitution or cancellation. It seems like the answer should be much simpler than those posted to the linked question.



Any ideas?



EDIT: I believe that this more general integral also vanishes:
$$int_1^{sqrt{t}}frac{1}{x}lnbigg(frac{t-sx^2+x^4}{tx-sx^2+x^3}bigg)dx=0$$










share|cite|improve this question
























  • @Rócherz Of course it isn't symmetric about $sqrt[3]{2}$; I simply mean that perhaps an appropriate substitution might lead to a cancellation.
    – Frpzzd
    Nov 20 at 0:47













up vote
11
down vote

favorite
5









up vote
11
down vote

favorite
5






5





In this question, the OP poses the following definite integral, which just happens to vanish:
$$int_1^sqrt2 frac{1}{x}lnbigg(frac{2-2x^2+x^4}{2x-2x^2+x^3}bigg)dx=0$$
As noticed by one commenter to the question, the only zero of the integrand is at $x=sqrt[3]{2}$, meaning that the integral of the integrand from $x=1$ to $x=sqrt[3]{2}$ is the additive inverse of the integral of the integrand from $x=sqrt[3]{2}$ to $x=sqrt{2}$.



This suggests some sort of symmetry obtainable by a substitution, but I cannot find an appropriate substitution or cancellation. It seems like the answer should be much simpler than those posted to the linked question.



Any ideas?



EDIT: I believe that this more general integral also vanishes:
$$int_1^{sqrt{t}}frac{1}{x}lnbigg(frac{t-sx^2+x^4}{tx-sx^2+x^3}bigg)dx=0$$










share|cite|improve this question















In this question, the OP poses the following definite integral, which just happens to vanish:
$$int_1^sqrt2 frac{1}{x}lnbigg(frac{2-2x^2+x^4}{2x-2x^2+x^3}bigg)dx=0$$
As noticed by one commenter to the question, the only zero of the integrand is at $x=sqrt[3]{2}$, meaning that the integral of the integrand from $x=1$ to $x=sqrt[3]{2}$ is the additive inverse of the integral of the integrand from $x=sqrt[3]{2}$ to $x=sqrt{2}$.



This suggests some sort of symmetry obtainable by a substitution, but I cannot find an appropriate substitution or cancellation. It seems like the answer should be much simpler than those posted to the linked question.



Any ideas?



EDIT: I believe that this more general integral also vanishes:
$$int_1^{sqrt{t}}frac{1}{x}lnbigg(frac{t-sx^2+x^4}{tx-sx^2+x^3}bigg)dx=0$$







integration definite-integrals symmetry






share|cite|improve this question















share|cite|improve this question













share|cite|improve this question




share|cite|improve this question








edited Nov 20 at 15:20

























asked Nov 20 at 0:42









Frpzzd

20.8k638104




20.8k638104












  • @Rócherz Of course it isn't symmetric about $sqrt[3]{2}$; I simply mean that perhaps an appropriate substitution might lead to a cancellation.
    – Frpzzd
    Nov 20 at 0:47


















  • @Rócherz Of course it isn't symmetric about $sqrt[3]{2}$; I simply mean that perhaps an appropriate substitution might lead to a cancellation.
    – Frpzzd
    Nov 20 at 0:47
















@Rócherz Of course it isn't symmetric about $sqrt[3]{2}$; I simply mean that perhaps an appropriate substitution might lead to a cancellation.
– Frpzzd
Nov 20 at 0:47




@Rócherz Of course it isn't symmetric about $sqrt[3]{2}$; I simply mean that perhaps an appropriate substitution might lead to a cancellation.
– Frpzzd
Nov 20 at 0:47















active

oldest

votes











Your Answer





StackExchange.ifUsing("editor", function () {
return StackExchange.using("mathjaxEditing", function () {
StackExchange.MarkdownEditor.creationCallbacks.add(function (editor, postfix) {
StackExchange.mathjaxEditing.prepareWmdForMathJax(editor, postfix, [["$", "$"], ["\\(","\\)"]]);
});
});
}, "mathjax-editing");

StackExchange.ready(function() {
var channelOptions = {
tags: "".split(" "),
id: "69"
};
initTagRenderer("".split(" "), "".split(" "), channelOptions);

StackExchange.using("externalEditor", function() {
// Have to fire editor after snippets, if snippets enabled
if (StackExchange.settings.snippets.snippetsEnabled) {
StackExchange.using("snippets", function() {
createEditor();
});
}
else {
createEditor();
}
});

function createEditor() {
StackExchange.prepareEditor({
heartbeatType: 'answer',
convertImagesToLinks: true,
noModals: true,
showLowRepImageUploadWarning: true,
reputationToPostImages: 10,
bindNavPrevention: true,
postfix: "",
imageUploader: {
brandingHtml: "Powered by u003ca class="icon-imgur-white" href="https://imgur.com/"u003eu003c/au003e",
contentPolicyHtml: "User contributions licensed under u003ca href="https://creativecommons.org/licenses/by-sa/3.0/"u003ecc by-sa 3.0 with attribution requiredu003c/au003e u003ca href="https://stackoverflow.com/legal/content-policy"u003e(content policy)u003c/au003e",
allowUrls: true
},
noCode: true, onDemand: true,
discardSelector: ".discard-answer"
,immediatelyShowMarkdownHelp:true
});


}
});














draft saved

draft discarded


















StackExchange.ready(
function () {
StackExchange.openid.initPostLogin('.new-post-login', 'https%3a%2f%2fmath.stackexchange.com%2fquestions%2f3005764%2fwhy-does-this-integral-equal-0%23new-answer', 'question_page');
}
);

Post as a guest















Required, but never shown






























active

oldest

votes













active

oldest

votes









active

oldest

votes






active

oldest

votes
















draft saved

draft discarded




















































Thanks for contributing an answer to Mathematics Stack Exchange!


  • Please be sure to answer the question. Provide details and share your research!

But avoid



  • Asking for help, clarification, or responding to other answers.

  • Making statements based on opinion; back them up with references or personal experience.


Use MathJax to format equations. MathJax reference.


To learn more, see our tips on writing great answers.





Some of your past answers have not been well-received, and you're in danger of being blocked from answering.


Please pay close attention to the following guidance:


  • Please be sure to answer the question. Provide details and share your research!

But avoid



  • Asking for help, clarification, or responding to other answers.

  • Making statements based on opinion; back them up with references or personal experience.


To learn more, see our tips on writing great answers.




draft saved


draft discarded














StackExchange.ready(
function () {
StackExchange.openid.initPostLogin('.new-post-login', 'https%3a%2f%2fmath.stackexchange.com%2fquestions%2f3005764%2fwhy-does-this-integral-equal-0%23new-answer', 'question_page');
}
);

Post as a guest















Required, but never shown





















































Required, but never shown














Required, but never shown












Required, but never shown







Required, but never shown

































Required, but never shown














Required, but never shown












Required, but never shown







Required, but never shown







Popular posts from this blog

Plaza Victoria

In PowerPoint, is there a keyboard shortcut for bulleted / numbered list?

How to put 3 figures in Latex with 2 figures side by side and 1 below these side by side images but in...